4 svar
1136 visningar
Nide är nöjd med hjälpen
Nide 114
Postad: 12 mar 2017 17:03 Redigerad: 12 mar 2017 17:55

Visa att en generaliserad integral divergerar

Uppgiften lyder såhär: Visa att integralen 01x4+1dx divergerar.

Detta är första gången som jag har stött på en generaliserad integral. Jag tänkte först att jag kunde bestämma den primitiva funktionen och sedan använda gränsvärden för att se om den går mot oändligheten eller inte. Tyvärr så är den primitiva funktionen för denna funktion ganska lång och svår att få fram (det får mig att tro att man inte ska lösa uppgiften med hjälp av den primitiva funktionen). Finns det något fiffigt sätt att bestämma om en integral divergerar/konvergerar utan att bestämma den primitiva funktionen?

Jag är dessutom lite extra förvirrad när det gäller svaret till denna uppgift. Jag har testat att lösa uppgiften med Wolfram Alpha (bara för att se hur Wolfram Alpha löser uppgiften) och Wolfram Alpha kom fram till att integralen istället KONVERGERAR! Så vem har rätt? Har min lärare skrivit fel i uppgiften?


Tråd flyttad från Matematik > Bevis till Matematik > Högskola av moderator då detta inte är ett komplett bevis. /statement

Hondel 1294
Postad: 12 mar 2017 18:32 Redigerad: 12 mar 2017 18:36

Ja, jag skulle säga att den är konvergent. Dock är jag inte helt säker på hur jag ska förklara det här om det är första gången du stöter på generaliserade integraler, första steget är ju att lära sig att räkna ut värdet på den mha gränsvärden och se om gränsvärdet är ändligt eller ej.

Men ja, det finns sätt att se om en integral är divergent/konvergent utan att räkna ut själva primitiven. Man använder jämförelser. Du ser ju att integralen från 0 till 1 är uppenbart konvergent (funktionen är ju definierad överallt). Så kvar har du en integral från 1 till  oändligheten. Då kan man använda en jämförelse. För x som är 1 och större gäller ju att 1x4+11x4

Då finns en jämförelsesats som säger (ungefär, jag glömmer säkert några detaljer) att om  f(x)g(x) för alla x i intervallet a till b, så gäller att om abg(x)dx är konvergent kommer också abf(x)dx

Eftersom 11x4dx är konvergent (det finns en sats för det) kommer också 11x4+1dx vara konvergent. Så därför är hela din integral konvergent

 

Edit: Jag kollade lite, och jag glömde ju såklart att nämna detaljer som måste vara uppfyllda (de är dock uppfyllda i ditt fall, men jag borde kanske rett ut det). Du kan läsa mer här om generaliserade integraler. Det är sats 1a jag pratar om, och jag vet att 1/x^4 konvergerar på intervallet 1 till oändligheten tack vara det som står i början under rubriken viktiga generaliserade integraler (p=4>1 i ditt fall)

Albiki 5096 – Fd. Medlem
Postad: 12 mar 2017 19:37

Hej!

Du kan dela upp integralen i två termer: en som handlar om små x-värden och en som handlar om stora x-värden.

    011x4+1dx+111+x4dx. \displaystyle \int_{0}^{1}\frac{1}{x^4+1}\,dx + \int_{1}^{\infty}\frac{1}{1+x^4}\,dx.

Om x x är ett litet positivt tal (mellan 0 och 1) så är (1+x4) (1+x^4) ett tal som ligger mellan 1 1 och 2 2 . Det betyder att kvoten 11+x4 \frac{1}{1+x^4} ligger mellan 0.5 0.5 och 1 1 , vilket i sin tur medför att kvotens integral ligger mellan 0.5 0.5 och 1 1 också.

    0.511+x41    0.5011x4+1dx1.\displaystyle 0.5 \leq \frac{1}{1+x^4} \leq 1 \quad \Rightarrow \quad 0.5 \leq \int_{0}^{1}\frac{1}{x^4+1}\,dx \leq 1.

Om x x är ett stort positivt tal (större än 1) så är (1+x4) (1+x^4) ungefär samma sak som talet x4 x^4 ; det är litet större än x4 x^4 . Det betyder att kvoten 11+x4 \frac{1}{1+x^4} är litet mindre än kvoten 1x4 \frac{1}{x^4} , vilket i sin tur medför att kvotens integral är mindre än integralen av kvoten 1x4 \frac{1}{x^4} .

    11+x41x4    11x4+1dx11x4dx=[13x3]1=13.\displaystyle \frac{1}{1+x^4} \leq \frac{1}{x^4} \quad \Rightarrow \quad \int_{1}^{\infty}\frac{1}{x^4+1}\,dx \leq \int_{1}^{\infty}\frac{1}{x^4}\,dx = [\frac{1}{3x^{3}}]_{\infty}^{1} = \frac{1}{3}.

Du ser att den generaliserade integralen ligger mellan talen 0.5 0.5 och 4/3 4/3 .

    0.5011x4+1dx+11x4+1dx1+13 .\displaystyle 0.5 \leq \int_{0}^{1}\frac{1}{x^4+1}\,dx + \int_{1}^{\infty}\frac{1}{x^4+1}\,dx \leq 1+\frac{1}{3}\ .

Albiki

Albiki 5096 – Fd. Medlem
Postad: 12 mar 2017 19:40

 Välkommen till PluggAkuten Nide!

Albiki

Nide 114
Postad: 12 mar 2017 20:06

Tack Albiki och Hondel! :)

Svara Avbryt
Close